Matemática, perguntado por joselitoluiz31, 2 meses atrás

3. Observe a tabela de reabastecimento do reservatório da bomba de combustível de um posto de gasolina. Reabastecimento do reservatório da bomba (litros) + 1350 +1300 Quantidade de carros 8 12 15 12 10 13 11 Reabastecimento de cada carro (litros) - 15 - 20 -25 - 30 - 40 - 45 -50 Fonte: dis para fins O valor positivo representa o quanto de gasolina foi colocado no reservatório dessa bomba e os valores negativos representam a quantidade retirada de gasolina do reservatório dessa bomba. Qual é a quantidade de gasolina existente nessa bomba, após os reabastecimentos de todos os veículos? AL P​


arthuralvescordeiro1: opa
arthuralvescordeiro1: mo dificil essas atividades

Soluções para a tarefa

Respondido por carvalhoelida06
11

Resposta:2.650

Explicação:espero ter ajudado


ratatarato29: podia ter colocado a conta
silvasantoseliseu63: podia ter colocado a conta né
Respondido por dugras
13

A quantidade de gasolina existente nessa bomba, após os reabastecimentos de todos os veículos é de 20 litros.

Operações com números inteiros

A tabela descrita é a seguinte:

\left[\begin{array}{ccc}Reabastecimento~ do~ reservat\'orio &Quantidade~ de ~carros&Reabastecimento ~de  \\da~ bomba ~(litros)& &cada~ carro~ (litros)\\+1350&8&-15\\&12&-20\\&15&-25\\&12&-30\\+1300&10&-40\\&13&-45\\&11&-50\end{array}\right]

Então somaremos os elementos da primeira coluna com o produto dos elementos da segunda coluna pelos elementos da terceira coluna, da sequinte maneira:

G = +1350 + 8 · (-15) + 12 · (-20) + 15 · (-25) + 12 · (-30) + 1300 + 10 · (-40) + 13 · (-45) + 11 · (-50)

Usando as regras de prioridades das expressões numéricas, efetuamos primeiro as multiplicações:

G = +1350 - 120 - 240 - 375 - 360 + 1300 - 400 - 585 - 550

Um modo mais simples de somarmos os inteiros é somarmos primeiro os positivos, depois os negativos e por fim subtraímos um do outro ficando o sinal do de maior valor absoluto:

G = 2650 - 2630 = 20 litros

Veja mais sobre operações com números inteiros em:

https://brainly.com.br/tarefa/51243635

#SPJ2

Perguntas interessantes